Jump to content

Poseidont's Content

There have been 336 items by Poseidont (Search limited from 10-06-2020)



Sort by                Order  

#289985 Xác định vị trí của d để SAMN min và AM+AN min

Posted by Poseidont on 24-12-2011 - 21:43 in Hình học

Cho hình bình hành ABCD, AB=5cm,AC=3,2 cm. d là dt đi qua đỉnh c cắt tia AB ở M,AD ở N
M: Xác định vị trí của d để SAMN min và AM+AN min



#301573 x+y-z=R+r

Posted by Poseidont on 29-02-2012 - 16:53 in Hình học

Cho tam giác ABC tù gọi R là bán kính đường tròn ngoại tiếp, r là bán kính đường tròn nội tiếp tam giác và x,y,z thứ tự là khoảng cách từ tâm O của đường tròn ngoại tiếp tới các cạnh BC,CA,AB.CMR x+y-z=R+r
Attached File  untitled3.bmp   947.02KB   47 downloads



#319292 Vì sao 1 + 1 = 2 ?

Posted by Poseidont on 25-05-2012 - 10:05 in Toán học lý thú

theo em mình đừng suy nghĩ về những vấn đề này mà nên đi vào vấn đề thực tiện, ok



#339902 Tuyển tập một số bài phương trình, hệ phương trình thi HSG tỉnh

Posted by Poseidont on 25-07-2012 - 09:17 in Phương trình - hệ phương trình - bất phương trình

Bài 74/Giải hệ phương trình
$$\left\{\begin{matrix} x^{19}+y^5=1890z+z^{2001}\\ y^{19}+z^5=1890x+x^{2001}\\z^{19}+x^5=1890y+y^{2001} & & \end{matrix}\right.$$
Bài 75
$\sqrt{\frac{a-bx}{cx}}=\frac{(b+c)x+x^2}{a+x^2}$ (với $a,b,c>0$)
____________
Đề này của tỉnh, thành phố nào????



#337062 Tuyển cầu thủ , thành lập VMF F.C

Posted by Poseidont on 17-07-2012 - 21:43 in Góc giao lưu

Sao không phải là 1 trong 2 đội sẽ thua :wub: :ukliam2:



#303341 Trí thông minh bẩm sinh và con đường trở thành một HSG toán!

Posted by Poseidont on 10-03-2012 - 14:36 in Kinh nghiệm học toán

trí thông minh chỉ làm con người ta nhạy ben hơn còn chủ yếu vẫn là sự chăm chỉ, sự nô lực của bản thân, mình đã từn là người học kém nhất lớp,nhưng nhờ có lòng quyết tâm nay đã vào đội tuyển toán



#319297 Topic phương trình, hệ phương trình vô tỉ

Posted by Poseidont on 25-05-2012 - 10:31 in Phương trình, hệ phương trình và bất phương trình

Bài 19: Giải phương trình:
$$7x^2+7x=\sqrt{\frac{4x+9}{28}}$$ với $x>0$

Cách giải khác Đặt $\sqrt{\frac{4x+9}{28}}=y+\frac{1}{2},,(y\geq \frac{-1}{2})$
Ta có $\frac{4x+9}{28}=y^2+y+\frac{1}{4}\Leftrightarrow 7y^2+7y=x+\frac{1}{2}$
Cùng vs phương trình ban đầu ban đầu ta có hệ
$$\left\{\begin{matrix} 7x^2+7x=y+\frac{1}{2} & \\ 7y^2+7y=x+\frac{1}{2}& \end{matrix}\right.$$
Đến đây đơn giản rồi nhé



#327466 Topic phương trình, hệ phương trình vô tỉ

Posted by Poseidont on 20-06-2012 - 23:42 in Phương trình, hệ phương trình và bất phương trình

Một bài nhẹ
Bài 83 Giải phương trình
$(2x-1)^2=12\sqrt{x^2-x-2}+1$



#316057 Topic luyện thi vào lớp 10 năm 2013 – 2014 (Hình học)

Posted by Poseidont on 12-05-2012 - 21:40 in Hình học

Bài 54 Cho $\vartriangle ABC$ có 3 góc nhọn nội tiếp $(O)$. $BE, CF$ là các đường cao, các tiếp tuyến với đường tròn $(O)$ tại $B;C$ cắt nhau tại $S$. Các đường thẳng $ BC, OS $ cắt nhau tại $M$
a/CMR $\frac{AB}{AE}=\frac{SB}{ME}$
b/ CMR $\vartriangle AEM\sim \vartriangle ABS$
c/ Gọi $N$ là giao điểm của $AM$ và $EF$ , $P$ là giao điểm của $SA$ và $BC$.CMR $NP \perp BC $



#317207 Topic luyện thi vào lớp 10 năm 2013 – 2014 (Hình học)

Posted by Poseidont on 16-05-2012 - 23:18 in Hình học

Bài 63 :
Cho tam giác ABC nhọn nội tiếp (O ; R) (AB < AC), các đườbnngg cao AD, BE, CF cắt nhau tại H.
a) Cm : BDHF, BFEC nội tiếp.
b) Cm : H là tâm đường tròn nội tiếp tam giác DEF.
c) Cm : $OA\perp EF$ và $\frac{S_{DEF}}{S_{ABC}}=\frac{r}{R}$ (với r là bk đường tròn nội tiếp tam giác DEF).
d) EF cắt BC tại M. Cm : $\frac{1}{CM}+\frac{1}{CD}=\frac{2}{BC}

d/$\frac{1}{CM}+\frac{1}{CD}=\frac{2}{BC}\Leftrightarrow \frac{1}{BC}-\frac{1}{CM}=\frac{1}{CD}-\frac{1}{CB}\Leftrightarrow \frac{MB}{BC.CM}=\frac{BD}{CD.CB}\Leftrightarrow \frac{MB}{BD}=\frac{CM}{CD}$
Ta lại có FB là tia phân giác của tam giác MFD ( cái này dễ bạn tự chứng minh nhé)
$\Rightarrow \frac{MF}{FD}=\frac{MB}{BD}$(1)
Mặt khác FC vuông góc với FB nên FC là tia phân giác ngoài
$\Rightarrow \frac{MF}{FD}=\frac{MC}{CD}$(2)
Từ (1) và (2) ta có điều phải chưng minh



#314730 Topic luyện thi vào lớp 10 năm 2013 – 2014 (Hình học)

Posted by Poseidont on 06-05-2012 - 18:10 in Hình học

Bài 29:
a/ E là trung điểm của dây MN $\Rightarrow OE\perp MN$ Mặt khác $AC\perp OC$
$\Rightarrow$ $A.E.O.C$ thuộc một đường tròn (cùng năm trên đương kính AO
b/ Tứ giác AEOC nội tiếp $\Rightarrow \widehat{AEC}=\widehat{AOC}=\frac{1}{2}sd BC$ (1)
Ta lại có $\widehat{BIC}=\frac{1}{2}sd BC$(2)
Từ (1) và(2) $\Rightarrow \widehat{AEC}=\widehat{BIC}$
c/$\widehat{AEC}=\widehat{BIC}$$\Rightarrow BI//MN$ (hai góc ở vị tri đồng dạng)



#316955 Topic luyện thi vào lớp 10 năm 2013 – 2014 (Hình học)

Posted by Poseidont on 16-05-2012 - 01:16 in Hình học

BÀI 58: Cho (O;R) và một đường thẳng d cố định không cắt (O).M là điểm di động trên d.Từ M kẻ tiếp tuyến MP và MQ với đường tròn..Hạ OH^d tại H và dây cung PQ cắt OH tại I;cắt OM tại K.
1. CMR: Tứ giác MHIK nội tiếp.
2. CMR: ${\rm{OJ}}.{\rm{OH}} = {\rm{OK}}.{\rm{OM}} = {{\rm{R}}^{\rm{2}}}$
3. CMr khi M di động trên d thì vị trí của I luôn cố định.
----------

Ta có MKHI là tứ giác nội tiếp
$\Rightarrow OI.OH=OK.OM=OP^2=R^2\Rightarrow OI=\frac{R^2}{OH}$ ( vì đường thẳng ố định nên OH cố định) Suy ra I cố định



#314928 Topic luyện thi vào lớp 10 năm 2013 – 2014 (Hình học)

Posted by Poseidont on 07-05-2012 - 18:51 in Hình học

Posted Image.
d/ Ta có CO//BF ( tự chứng minh nhé)
O là trung điểm của AB
$\Rightarrow CF=CA$
MI//FC$\Rightarrow \frac{MI}{FC}=\frac{IB}{BC}$(1)(Ta lét)
IH//CA$\Rightarrow \frac{IB}{BC}=\frac{IH}{AC}$(2)
Từ (1) và(2) $\Rightarrow \frac{IM}{FC}=\frac{IH}{AC}$
$\Rightarrow IM=IC$
$\Rightarrow{S_{\Delta AIM}}+{S_{\Delta BIM}}=\frac{1}{2}.MI.(AH+HB)=\frac{1}{2}MI.AB=\frac{1}{2}IH.AB={S_{\Delta AIB}}$



#317197 Topic luyện thi vào lớp 10 năm 2013 – 2014 (Hình học)

Posted by Poseidont on 16-05-2012 - 22:56 in Hình học

Bài 64: AA1 là đường cao của tam giác ABC , H là trực tâm của tam giác đó.P là điểm tùy ý nằm trên đường tròn ngoại tiếp tam giác. Q là điểm được lấy trên PH sao cho $HP.HQ=AH.HA1$.CMR Q nằm trên đường tròn Euler của tam giác ABC



#324338 Topic luyện thi vào lớp 10 năm 2013 – 2014 (Hình học)

Posted by Poseidont on 12-06-2012 - 10:18 in Hình học

Bài 112.
Cho tam giác ABC nhọn nội tiếp đường tròn (O;R). 3 đường cao AD, BE, CF cắt nhau tại H. AD cắt (O) tại K.
Tính $AD^{2}+BD^{2}+CD^{2}+DK^{2} theo R$

TheoPythagore$\Rightarrow AD^{2}+BD^{2}+CD^{2}+DK^{2} =AB^2+CK^2$
Kẻ đường kính BI ta có IC//AK $\Rightarrow CK=AI$
BI là đường kính nên tam giác ABI vuông tại I áp dụng Pythagore $AB^2+AI^2=(2R)^2\Rightarrow AB^2+CK^2=4R^2$
Nên $AD^{2}+BD^{2}+CD^{2}+DK^{2}=4R^2$



#322462 Topic luyện thi vào lớp 10 năm 2013 – 2014 (Hình học)

Posted by Poseidont on 04-06-2012 - 20:13 in Hình học

Bài 105: Cho (O) đường kính AB , M là điểm đối xứng với O qua A, đường thẳng d đi qua M cắt (O) tại C và D (C nằm giữa M và D), AD cắt BC tại I.CMR tam giác IOA cân
:lol: :namtay >:)



#318019 Topic bất đẳng thức THCS (2)

Posted by Poseidont on 19-05-2012 - 23:17 in Bất đẳng thức và cực trị

BÀI 359:
Cho a,b,c không âm thỏa a+b+c=3. Chứng minh rằng:
$\frac{a}{{b + {c^2}}} + \frac{b}{{c + {a^2}}} + \frac{c}{{a + {b^2}}} \ge \frac{3}{2}$
-----------

Một lời giải khá dài cho bài này, và có thể bị sai sót nữa, mọi người kiểm chứng dùm nha.

Theo BĐT Cauchy-Schwar ta có:

$VT\geq \frac{(a+b+c)^2}{ab+bc+ca+a^2b+b^2c+c^2a}=\frac{9}{(ab+bc+ca-abc)+(a^2b+b^2c+c^2a+abc)}$

Sử dụng BĐT quen thuộc với $a+b+c=3$ thì $$a^2b+b^2c+c^2a+abc\leq 4$$

Ta sẽ chứng minh $$A=ab+bc+ca-abc\leq 2$$

Thật vậy $$A=b(a+c)+ac(1-b)\leq b(3-b)+\frac{(a+c)^2(1-b)}{4}=b(3-b)+\frac{(3-b)^2(1-b)}{4}=\frac{(3-b)(b^2+3)}{4}$$

Đạo hàm $A'=\frac{-3(b-1)^2}{4}$

$$A'=0 \Leftrightarrow b=1$$

Dựa vào BBT ta tìm được $A\leq 2$ khi $a=b=c=1$

Từ đây, thay vào BĐT ban đầu, ta được:

$$VT\geq \frac{9}{(ab+bc+ca-abc)+(a^2b+b^2c+c^2a+abc)}\geq \frac{9}{2+4}=\frac{3}{2}$$

Dấu bằng xảy ra khi $a=b=c=1$




#293491 Topic bất đẳng thức THCS (2)

Posted by Poseidont on 12-01-2012 - 15:51 in Bất đẳng thức và cực trị

Bài 52:
Cho $x,y,z>0$
CMR: \[16xyz\left( {x + y + z} \right) \le 3\sqrt[3]{{{{\left( {x + y} \right)}^4}{{\left( {z + y} \right)}^4}{{\left( {x + z} \right)}^4}}}\]

$$(x+y)(y+z)(z+x)=(x+y+z)(xy+yz+zx)-xyz\geq 3(x+y+z)\sqrt[3]{x^{2}y^{2}z^{2}}-\dfrac{1}{3}(x+y+z)\sqrt[3]{x^{2}y^{2}z^{2}}=\dfrac{8}{3}(x+y+z)\sqrt[3]{x^{2}y^{2}z^{2}}$$
áp dụng bdt trên
$\Rightarrow 3\sqrt[3]{{{{\left( {x + y} \right)}^4}{{\left( {z + y} \right)}^4}{{\left( {x + z} \right)}^4}}}\geq 3\cdot \dfrac{8}{3}(x+y+z)\sqrt[3]{x^{2}y^{2}z^{2}}\cdot$$\sqrt[3]{\dfrac{8}{3}(x+y+z)\sqrt[3]{x^{2}y^{2}z^{2}}\cdot}$$\geq 8(x+y+z)\sqrt[3]{x^{2}y^{2}z^{2}}\cdot \sqrt[3]{8xyz}=16(x+y+z)xyz$
$\Rightarrow dpcm$



#294117 Topic bất đẳng thức THCS (2)

Posted by Poseidont on 16-01-2012 - 09:19 in Bất đẳng thức và cực trị

Bài 77:
Gọi $a,b,c$ là độ dài cạnh của 1 tam giác có 3 góc nhọn.
CMR: \[\forall x,y,z \in R\]
Ta luôn có: \[\frac{{{x^2}}}{{{a^2}}} + \frac{{{y^2}}}{{{b^2}}} + \frac{{{z^2}}}{{{c^2}}} > \frac{{2{x^2} + 2{y^2} + 2{z^2}}}{{{a^2} + {b^2} + {c^2}}}\]


P/s: Đây là đề thi Chuyên Lam Sơn Thanh Hóa nhá.

áp dụng bdt S vac
$\frac{{{x^2}}}{{{a^2}}} + \frac{{{y^2}}}{{{b^2}}} + \frac{{{z^2}}}{{{c^2}}}\geq \frac{(x+y+z)^{2}}{a^{2}+b^{2}+c^{2}}=\frac{x^{2}+y^{2}+z^{2}+x(y+z)+y(x+z)+z(x+y)}{a^{2}+b^{2}+c^{2}}> \frac{2x^{2}+2y^{2}+2z^{2}}{a^{2}+b^{2}+c^{2}}$
(theo bdt tam giac ta co x+y>z, ....)
DPCM



#298732 Topic bất đẳng thức THCS (2)

Posted by Poseidont on 09-02-2012 - 16:54 in Bất đẳng thức và cực trị

Bài 261: Cho x,y,z $ \ge 0;x + y + z \le 3$
CMR: $$\frac{x}{{1 + x^2 }} + \frac{y}{{1 + y^2 }} + \frac{z}{{1 + z^2 }} \le \frac{3}{2} \le \frac{1}{{1 + x}} + \frac{1}{{1 + y}} + \frac{1}{{1 + z}}$$
Bài 262: Cho a,b,c là 3 số thực dương thay đổi thỏa mãn a+b+c=3
CMR: $$A = \frac{{a^3 }}{{b(2c + a)}} + \frac{{b^3 }}{{c(2a + b)}} + \frac{{c^3 }}{{a(2b + c)}} \ge 1$$
Bài 263: Cho a,b,c là 3 số thực dương. CMR
$$\frac{{b + c}}{{a + \sqrt[3]{{4(b^3 + c^3 )}}}} + \frac{{a + c}}{{b + \sqrt[3]{{4(a^3 + c^3 )}}}} + \frac{{b + a}}{{c + \sqrt[3]{{4(b^3 + a^3 )}}}} \le 2$$
Bài 264: Cho a,b,c là các số thực dương thay đổi thỏa mãn ab+bc+ac=abc. CMR:
$$\sqrt {a + bc} + \sqrt {b + ac} + \sqrt {c + ab} \ge \sqrt {abc} + \sqrt a + \sqrt c + \sqrt b$$
Bài 265: Cho a,b,c là các số dương thay đổi thỏa mãn ab+bc+ac=1. CMR
$$\frac{1}{{ab}} + \frac{1}{{ac}} + \frac{1}{{cb}} \ge 3 + \sqrt {\frac{1}{{a^2 }} + 1} + \sqrt {\frac{1}{{b^2 }} + 1} + \sqrt {\frac{1}{{c^2 }} + 1}$$
Bài 266: CMR với mọi số thực dương a,b ta có BĐT sau
$$\frac{{a + b}}{2} \ge \sqrt {ab} + \frac{{(a - b)^2 (a + 3b)(b + 3a)}}{{16(a + b)^3 }}$$
THTT
Bài 267: Cho các số thực dương a,b,c thỏa mãn $3(ab+bc+ac)=1$
CMR: $$\frac{a}{{a^2 - bc + 1}} + \frac{b}{{b^2 - ac + 1}} + \frac{c}{{c^2 - ba + 1}} \ge \frac{1}{{a + b + c}}$$

bài 263,
áp dụng bdt $\frac{a^{3}+b^{3}}{2}\geqslant (\frac{a+b}{2})^{3}$
=>$\sqrt[3]{4(b^{3}+c^{3})}\geqslant b+c =>VT\leq 2$
Q.E.D
bài 265
VT=$\sum \frac{ab+bc+ca}{ab}=\sum 1+\frac{c}{a}+\frac{c}{b}$
VP=$3+\sum \sqrt{\frac{a^{2}+1}{a^{2}}}=3+\sum \sqrt{\frac{(a+b)(a+c)}{a^{2}}}\leq 3+\frac{(a+b)+(a+c)}{2a}=3+\frac{6+\sum \frac{b}{a}+\frac{c}{a}}{2}$
trừ hồi rồi dùng cô si là ra



#326936 Topic bất đẳng thức THCS (2)

Posted by Poseidont on 19-06-2012 - 10:09 in Bất đẳng thức và cực trị

Bài 387 ( Đối với những ngưòi chưa biết chuẩn hoá)
BĐT $\Leftrightarrow 4(a+b+c).\sum \frac{a}{(b+c)^2}\geq 9\Leftrightarrow \sum 4\frac{a^2}{(b+c)^2}+\sum 4\frac{a}{b+c}\geq 9$
Đến đây Áp dụng Cauchy Scwharz dạng Engel và Nesbit ta có điều phải chứng minh



#292005 Topic bất đẳng thức THCS (2)

Posted by Poseidont on 03-01-2012 - 23:18 in Bất đẳng thức và cực trị

Bài 19: Cho $a,b>1$. Tìm min của $E = \dfrac{{{a^2}}}{{b - 1}} + \dfrac{{{b^2}}}{{a - 1}}$

Bài 19

Áp dụng BĐT Cauchy ta có:
$\dfrac{a^{2}}{b-1}+4(b-1)\geq 2\sqrt{\dfrac{a^{2}}{b-1}\cdot4(b-1) }=4a$
Tương tự
$\dfrac{b^{2}}{a-1}+4(a-1)\geq4b$
$\Rightarrow VT\geq 4a+4b-4(b-1)-4(a-1)=8$
dấu $=$ xảy ra khi và chỉ khi $ a=b=2$



#293481 Topic bất đẳng thức THCS (2)

Posted by Poseidont on 12-01-2012 - 15:19 in Bất đẳng thức và cực trị

Bài 50:
Cho a,b,c thực dương thỏa $a+b+c=3$
CMR: $a^4+b^4+c^4\geq a^3+b^3+c^3$



Bài 50 nè
Áp dụng bdt Cauchy:
$a^{4}+a^{4}+a^{4}+1\geq 4a^{3}$
$b^{4}+b^{4}+b^{4}+1\geq 4b^{3}$
$c^{4}+c^{4}+c^{4}+1\geq 4c^{3}$
Giờ chỉ cần chứng minh $a^{3}+b^{3}+c^{3}\geq 3$
Ta có:
$a^{3}+1+1\geq 3a \Rightarrow a^{3}+b^{3}+c^{3}\geq 3$
vì a+b+c=3
$\Rightarrow dpcm$



#326648 Topic bất đẳng thức THCS (2)

Posted by Poseidont on 18-06-2012 - 11:55 in Bất đẳng thức và cực trị

Bài này đổi biến đuợc mà, em h đang mắc mọi ngưòi thông cảm nha
$\inline \sum \frac{a}{a^2+2b+3}\leq \frac{1}{2}\Leftrightarrow \sum \frac{a}{4a+b+1}\leq \frac{1}{2}\Rightarrow \sum \frac{4a}{4a+b+1}\leq 2\Rightarrow \sum \frac{b+1}{4a+b+1}\geq 1$
Đến đây $\sum \frac{b+1}{4a+b+1}\geq 1=\sum \frac{b}{4a+b+1}+\frac{1}{4a+b+1}$
Ta áp dụng Cauchy Scwharz và áp dụng cái $a^2+b^2+c^2=3$ là $\square$



#343156 Topic bất đẳng thức THCS (2)

Posted by Poseidont on 03-08-2012 - 20:28 in Bất đẳng thức và cực trị

Cách khác
Áp dụng Cauchy Schwarz và giả thiết
$x^2+xyz=x^2+xy+yz+xz=(x+y)(x+z)\geq (x+\sqrt{yz})^2\Rightarrow \sqrt{x(x+yz)}\geq x+\sqrt{yz}$
$\Rightarrow \sqrt{x+yz}\geq \frac{x+\sqrt{yz}}{\sqrt{x}}=\sqrt{x}+\frac{\sqrt{yz}}{\sqrt{x}}$
Xây dựng các bđt khác tuơng tự
Ta lại có
$\sum \frac{\sqrt{xy}}{\sqrt{z}}=\sqrt{xyz}(\frac{1}{x}+\frac{1}{y}+\frac{1}{z})=\sqrt{xyz}$
$\Rightarrow Q.E.D$